Prove a linear mapping of a polynomial function is a map

Click For Summary
The discussion centers on proving that the mapping Δ defined for polynomials is a linear mapping. Participants clarify that the correct condition for linearity is Δ[P(x) + P(y)] = Δ[P(x)] + Δ[P(y)], rather than the initially stated condition. The mapping involves performing differential operations on polynomials, and there is confusion regarding the application of derivatives. A participant emphasizes the importance of comparing polynomials of the same variable to demonstrate linearity effectively. The conversation concludes with a consensus on the correct approach to proving the linearity of the mapping.
sa1988
Messages
221
Reaction score
23

Homework Statement



The bane of all physicists... 'Proof' questions...

So we have the mapping,

Δ : P3→P3
Δ[P(x)] = (x2-1) d2P/dx2 + x dP/dx

And I need to prove that this is a linear mapping

Homework Equations



Linear maps must satisfy:
Δ[P(x+y)] = Δ[P(x)] + Δ[P(y)]
and
Δ[P(αx)] - αΔ[P(x)]

The Attempt at a Solution



I'm not sure what to do. I've tried working through the actual mapping, performing the differential operations on the polynomial:
ax3+bx2+cx+d
and on:
a(x+y)3+b(x+y)2+c(x+y)+d
then expanded the brackets to see if I could separate the x and y terms to show that it's possible to pull them apart and demonstrate equality with Δ[P(x)] + Δ[P(y)]
But it doesn't work. My best bet is that I've done something wrong regarding the part where I need to do
d2P/d(x+y)2. I've never really had to work in that way before. Maybe I've gone wrong completely.

Any advice?

Thanks! :oldsmile:
 
Physics news on Phys.org
sa1988 said:

Homework Statement



So we have the mapping,

Δ : P3→P3
Δ[P(x)] = (x2-1) d2P/dx2 + x dP/dx

And I need to prove that this is a linear mapping

Homework Equations



Linear maps must satisfy:
Δ[P(x+y)] = Δ[P(x)] + Δ[P(y)]
The above isn't right.
It should be Δ[P(x) + P(y)] = Δ[P(x)] + Δ[P(y)]
IOW, the Δ map carries a sum of functions to the sum of the transformations of the functions.
sa1988 said:
and
Δ[P(αx)] - αΔ[P(x)]

The Attempt at a Solution



I'm not sure what to do. I've tried working through the actual mapping, performing the differential operations on the polynomial:
ax3+bx2+cx+d
and on:
a(x+y)3+b(x+y)2+c(x+y)+d
then expanded the brackets to see if I could separate the x and y terms to show that it's possible to pull them apart and demonstrate equality with Δ[P(x)] + Δ[P(y)]
But it doesn't work. My best bet is that I've done something wrong regarding the part where I need to do
dP2/d(x+y)2. I've never really had to work in that way before. Maybe I've gone wrong completely.

Any advice?

Thanks! :oldsmile:
 
  • Like
Likes sa1988
Mark44 said:
The above isn't right.
It should be Δ[P(x) + P(y)] = Δ[P(x)] + Δ[P(y)]
IOW, the Δ map carries a sum of functions to the sum of the transformations of the functions.

Bingo, you've got it. I remember now.

Such a little thing - thanks for pointing it out!
 
Taking the differential on ## P(x) = Ax^3 + Bx^2 + Cx + D ## should give:
##\Delta P(x) = (x^2-1)(6Ax+2B)+x(3Ax^2+2Bx+C) = 9Ax^3 +4Bx^2+(C-6A)x-2B##
As Mark pointed out, you want to compare two polynomials ##P_1(x) = ax^3 + bx^2 + cx + d## and ##P_2(x) = hx^3 + ix^2 + jx + k##, not polynomials of different variables.
This should make sense, because a polynomial of greater than first order is known to not be a linear function.
 
  • Like
Likes sa1988
Mark44 said:
The above isn't right.
It should be Δ[P(x) + P(y)] = Δ[P(x)] + Δ[P(y)]
IOW, the Δ map carries a sum of functions to the sum of the transformations of the functions.

Shouldn't that be ##\Delta [P_1(x) + P_2(x)] = \Delta P_1(x) + \Delta P_2(x) \: \forall x##?
 
Ray Vickson said:
Shouldn't that be ##\Delta [P_1(x) + P_2(x)] = \Delta P_1(x) + \Delta P_2(x) \: \forall x##?
Now that you mention it, yes...
Thanks for the correction.
 
Question: A clock's minute hand has length 4 and its hour hand has length 3. What is the distance between the tips at the moment when it is increasing most rapidly?(Putnam Exam Question) Answer: Making assumption that both the hands moves at constant angular velocities, the answer is ## \sqrt{7} .## But don't you think this assumption is somewhat doubtful and wrong?

Similar threads

  • · Replies 5 ·
Replies
5
Views
2K
Replies
9
Views
2K
Replies
5
Views
2K
Replies
4
Views
2K
  • · Replies 4 ·
Replies
4
Views
2K
Replies
2
Views
2K
  • · Replies 1 ·
Replies
1
Views
4K
  • · Replies 23 ·
Replies
23
Views
2K
Replies
9
Views
3K
  • · Replies 2 ·
Replies
2
Views
2K